完整版真题免费下载
+答案解析请参考文末
Are there integers and such that and are both perfect cubes of integers?
Each cell of an board is filled with some nonnegative integer. Two numbers in the filling are said to be adjacent if their cells share a common side. (Note that two numbers in cells that share only a corner are not adjacent). The filling is called a garden if it satisfies the following two conditions:
(i) The difference between any two adjacent numbers is either or .
(ii) If a number is less than or equal to all of its adjacent numbers, then it is equal to .
Determine the number of distinct gardens in terms of and .
In triangle , points lie on sides respectively. Let , , denote the circumcircles of triangles , , , respectively. Given the fact that segment intersects , , again at respectively, prove that .
Let be the number of ways to write as a sum of powers of , where we keep track of the order of the summation. For example, because can be written as , , , , , and . Find the smallest greater than for which is odd.
Quadrilateral is inscribed in the semicircle with diameter . Segments and meet at . Point is the foot of the perpendicular from to line . Point lies on such that line is perpendicular to line . Let be the intersection of segments and . Prove that、
Find all real numbers satisfying
No, such integers do not exist. This shall be proven by contradiction, by showing that if is a perfect cube then cannot be.
Remark that perfect cubes are always congruent to , , or modulo . Therefore, if , then .
If , then note that . (This is because if then .) Therefore and , contradiction.
Otherwise, either or . Note that since is a perfect sixth power, and since neither nor contains a factor of , . If , thenSimilarly, if , thenTherefore , contradiction.
Therefore no such integers exist.
We shall prove that such integers do not exist via contradiction. Suppose that and for integers x and y. Rearranging terms gives and . Solving for a and b (by first multiplying the equations together and taking the sixth root) gives a = and b = . Consider a prime p in the prime factorization of and . If it has power in and power in , then - is a multiple of 24 and - also is a multiple of 24.
Adding and subtracting the divisions gives that - divides 12. (actually, is a multiple of 4, as you can verify if . So the rest of the proof is invalid.) Because - also divides 12, divides 12 and thus divides 3. Repeating this trick for all primes in , we see that is a perfect cube, say . Then and , so that and . Clearly, this system of equations has no integer solutions for or , a contradiction, hence completing the proof.
Therefore no such integers exist.
Let and . Then, , , andNow take (recall that perfect cubes and perfect sixth powers ) on both sides. There are cases to consider on what values that and take. Checking these cases, we see that only or yield a valid residue (specifically, ). But this means that , so socontradiction.
If is a perfect cube, then can be one of , so can be one of , , or . If were divisible by , we'd have , which we've ruled out. So , which means , and therefore .
We've shown that can be one of , so can be one of . None of these are possibilities for a perfect cube, so if is a perfect cube, cannot be.
As in previous solutions, notice . Now multiplying gives , which is only , so after testing all cases we find that . Then since , and (Note that cannot be ). Thus we find that the inverse of is itself under modulo , a contradiction.
We claim that any configuration of 's produces a distinct garden. To verify this claim, we show that, for any cell that is nonzero, the value of that cell is its distance away from the nearest zero, where distance means the shortest chain of adjacent cells connecting two cells. Now, since we know that any cell with a nonzero value must have a cell adjacent to it that is less than its value, there is a path that goes from this cell to the that is decreasing, which means that the value of the cell must be its distance from the as the path must end. From this, we realize that, for any configuration of 's, the value of each of the cells is simply its distance from the nearest , and therefore one garden is produced for every configuration of 's.
However, we also note that there must be at least one in the garden, as otherwise the smallest number in the garden, which is less than or equal to all of its neighbors, is , which violates condition . There are possible configurations of and not in the garden, one of which has no 's, so our total amount of configurations is
Note: "bordering" and "surrounding" mean that two cells have to touch on a side; one vertex is not good enough.
We note that there is no real step to begin the problem, so start by constructing the base case: put ALL the zeroes into the board. Then note that all squares bordering it (corners alone don't count) have to be . After doing that, what can a cell bordering a 1 have as its value? Either 1 or 2, based on (i). But if it were 1, then all cells surrounding it would have at least 1 as their value by (i). And by (ii) the value would have to be 0, which contradicts our initial construction. Therefore, all the cells bordering a 1 have to be 2. Looks too simple for JMO, right? We apply the same logic to a cell bordering value : either it is or . If it is , however, by constraint (i) and (ii) we realize that this cell has to be 0 (because neighbors cannot be less than it), contradicting our construction again! Therefore, that cell has to be . And so on until the grid is filled. Basically the problem reduces to finding all 0s, surrounding them with 1s, and surrounding the 1s with 2s, etc. Beautiful! I have established a bijection between the zeros placed in the grid and the arrangement, therefore there are solutions right? NO! Take the smallest cell. It has to be via criterion (ii). So a case with zero zeros is apocryphal. Our answer is .
In this solution, all lengths and angles are directed.
Firstly, it is easy to see by that concur at a point . Let meet again at and , respectively. Then by Power of a Point, we haveThuslyBut we claim that . Indeed,andTherefore, . Analogously we find that and we are done.
Diagram Refer to the Diagram link.
By Miquel's Theorem, there exists a point at which intersect. We denote this point by Now, we angle chase:In addition, we haveNow, by the Ratio Lemma, we have(by the Law of Sines in )(by the Law of Sines in )by the Ratio Lemma. The proof is complete.
Use directed angles modulo .
Lemma.
Proof.
Now, it follows that (now not using directed angles)using the facts that and , and are similar triangles, and that equals twice the circumradius of the circumcircle of .
First of all, note that = where is the largest integer such that . We let for convenience.
From here, we proceed by induction, with our claim being that the only such that is odd are representable of the form
We induct on . It is trivially true for and . From here, we show that, if the only numbers where is odd are of the form described above, then the only numbers that are odd are of that form. We first consider all numbers , such that , going from the lower bound to the upper bound (a mini induction, you might say). We know that . For a number in this summation to be odd, . However, we know that , so must be equal to , or else cannot be in that interval. Now, from this, we know that , as . Therefore, and are distinct, and thus and are odd; since there are just two odd numbers, the ending sum for any is even. Finally, considering , the only odd number is , so the ending sum is odd.
The smallest greater than expressible as is
Of course, as with any number theory problem, use actual numbers to start, not variables! By plotting out the first few sums (do it!) and looking for patterns, we observe that , where represents the largest power of that is smaller than . I will call this sum the Divine Sign, or DS.
But wait a minute... we are trying to determine odd/even of . Why not call all the evens 0 and odds 1, basically using mod 2? Sounds so simple. Draw a small table for the values: as goes up from , you get: . We have to set for this to work. Already it looks like is only odd if .
The only tool here is induction. The base case is clearly established. Then let's assume we successfully made our claim up to . We need to visit numbers from to . Realize that has for because there will be two numbers in DS that give a of one: and .
But to look at whether a value of is 1 or 0, we need to revisit our first equation. We can answer this rather natural question: When will a number to be inducted upon, say , ever have a 1 as in the DS equation? Well- because by our assumption of the claim up to , we know that the only way for that to happen is if in the DS is equal to . Clearly .
Finally, we can simplify. Using our last equation, , regrouping gives .
Most importantly, realize that can be from to , because of the restraints on mentioned earlier. Same with . Immediately at least one of and has to be . If both were smaller, LHS is greater, contradiction. If both were greater, RHS is greater, contradiction.
Therefore, by setting one of or to , we realize .
The conclusion is clear, right? Each from to yields two distinct cases: one of and is equal to , while the other is LESS THAN . But for , there is ONE CASE: BOTH values have to equal . Therefore, the only that has as odd must only be , because the other ones yield a of 1+1=0 in our mod. That proves our induction for a new power of 2, namely , meaning that is only odd if , and we are almost done...
Thus, the answer is .
This was pretty intuitive, and realizing quite steps about how powers of 2 work gave us the solution! Estimated time: ~50 minutes. Cheers- expiLnCalc.
Let us use coordinates. Let O, the center of the circle, be (0,0). WLOG the radius of the circle is 1, so set Y (1,0) and X (-1,0). Also, for arbitrary constants and set A and B . Now, let's use our coordinate tools. It is easily derived that the equation of is and the equation of is , where and are defined appropriately. Thus, by equating the y's in the equation we find the intersection of these lines, , is . Also, . It shall be left to the reader to find the slope of , the coordinates of Q and C, and use the distance formula to verify that .
First of all
since the quadrilateral is cyclic, and triangle is rectangle, and is orthogonal to . Now
because is cyclic and we have proved that
so is parallel to , andNow by Ptolomey's theorem on we havewe see that triangles and are similar sinceandis already proven, soSubstituting yieldsdividing by We getNow triangles , and are similar sobut also triangles and are similar and we getComparing we have,Substituting,Dividing the new relation by and multiplying by we getbutsince triangles and are similar, becauseandsince Substituting again we getNow since triangles and are similar we haveand by the similarity of and , we getso substituting, and separating terms we getIn the beginning we prove that and so
It is obvious thatfor some value . Also, note that . SetWe haveandThis givesSimilarly, we can deduce thatAdding gives
First, since is the diameter and , , and lie on the circle,. Next, because and are both perpendicular to , we have to be parallel to .
Now looking at quadrilateral , we see that this is cyclic becauseSet , and . Now,since and are parallel. Also,That meanssoThis means , so and are parallel. Finally, we can look at the equation. We knowso We also knowso Plugging this into the LHS of the equation, we getNow, let be the point on such that is perpendicular to . Also, since , their arcs have equal length, and . Now, the LHS is simplified even more towhich is equal towhich is equal toThis completes the proof.
The key Lemma is:for all . Equality holds when .
This is proven easily.by Cauchy. Equality then holds when .
Now assume that . Now note that, by the Lemma,
. So equality must hold. So and . If we let , then we can easily compute that . Now it remains to check that .
But by easy computations, , which is obvious. Also , which is obvious, since .
So all solutions are of the form , and all permutations for .
Remark: An alternative proof of the key Lemma is the following: By AM-GM,. Now taking the square root of both sides gives the desired. Equality holds when .
Without loss of generality, let . Then .
Suppose x = y = z. Then , so . It is easily verified that has no solution in positive numbers greater than 1. Thus, for x = y = z. We suspect if the inequality always holds.
Let x = 1. Then we have , which simplifies toand henceLet us try a few examples: if y = z = 2, we have ; if y = z, we have , which reduces to . The discriminant (16 - 20) is negative, so in fact the inequality is strict. Now notice that yz - y - z + 3 = (y-1)(z-1) + 2. Now we see we can let ! Thus,and the claim holds for x = 1.
If x > 1, we see the will provide a huge obstacle when squaring. But, using the identity :which leads toAgain, we experiment. If x = 2, y = 3, and z = 3, then .
Now, we see the finish: setting gives . We can solve a quadratic in u! Because this problem is a #6, the crown jewel of USAJMO problems, we do not hesitate in computing the messy computations:
Because the coefficient of is positive, all we need to do is to verify that the discriminant is nonpositive:
Let us try a few examples. If y = z, then the discriminant D = .
We are almost done, but we need to find the correct argument. (How frustrating!) Success! The discriminant is negative. Thus, we can replace our claim with a strict one, and there are no real solutions to the original equation in the hypothesis.
--Thinking Process by suli
WLOG, assume that . Let and . Then , and . The equation becomesRearranging the terms, we haveTherefore and Express and in terms of , we have and Easy to check that is the smallest among , and Then , and Let , we have the solutions for as follows: and permutations for all
© 2024. All Rights Reserved. 沪ICP备2023009024号-1